Bounding number of solutions for floor function equation
by Ciobi_, Apr 2, 2025, 12:39 PM
Let
be a positive integer. Consider the following equation:
a) For
, solve the given equation in
.
b) Prove that, for any
, the equation has at most
real solutions.

![\[ \{x\}+\{2x\}+ \dots + \{nx\} = \lfloor x \rfloor + \lfloor 2x \rfloor + \dots + \lfloor 2nx \rfloor\]](http://latex.artofproblemsolving.com/8/2/9/8299b3ceb1650418c0a373536cb1d2931900efd9.png)


b) Prove that, for any


Normal but good inequality
by giangtruong13, Mar 31, 2025, 4:04 PM
Function on positive integers with two inputs
by Assassino9931, Jan 27, 2025, 10:03 AM
The function
is such that
for any positive integers
. Assume there exists a positive integer
such that
for all positive integers
. Determine all possible values of
.







This post has been edited 1 time. Last edited by Assassino9931, Jan 27, 2025, 10:06 AM
Problem 1
by SpectralS, Jul 10, 2012, 5:24 PM
Given triangle
the point
is the centre of the excircle opposite the vertex
This excircle is tangent to the side
at
, and to the lines
and
at
and
, respectively. The lines
and
meet at
, and the lines
and
meet at
Let
be the point of intersection of the lines
and
, and let
be the point of intersection of the lines
and
Prove that
is the midpoint of 
(The excircle of
opposite the vertex
is the circle that is tangent to the line segment
, to the ray
beyond
, and to the ray
beyond
.)
Proposed by Evangelos Psychas, Greece























(The excircle of







Proposed by Evangelos Psychas, Greece
APMO 2012 #3
by syk0526, Apr 2, 2012, 3:09 PM
Determine all the pairs
of a prime number
and a positive integer
for which
is an integer.




nice system of equations
by outback, Oct 8, 2008, 2:41 PM
Two circles, a tangent line and a parallel
by Valentin Vornicu, Oct 24, 2005, 10:15 AM
Two circles
and
intersect at two points
and
. Let
be the line tangent to these circles at
and
, respectively, so that
lies closer to
than
. Let
be the line parallel to
and passing through the point
, with
on
and
on
. Lines
and
meet at
; lines
and
meet at
; lines
and
meet at
. Show that
.



























Number theory or function ?
by matematikator, Mar 18, 2005, 2:10 PM
Does there exist a function
such that if
and
are distinct rational numbers satisfying
or
, then
? Justify your answer.
Proposed by Dan Brown, Canada






Proposed by Dan Brown, Canada
This blog reflects my thoughts on the mathematics that I grapple with. Hopefully these rumblings could be organized as to be palatable to a mathematical audience.
Archives














Tags
About Owner
- Posts: 866
- Joined: Jan 12, 2011
Blog Stats
- Blog created: Mar 9, 2011
- Total entries: 42
- Total visits: 27216
- Total comments: 24
Search Blog